LE1 Flashcards

1
Q

Baby Boy M was born full term. Labor was uneventful. He was given breast milk. He was apparently well when he was noted to have yellowish discoloration of the skin on the 30th hour of life. No associated symptoms noted. Total serum bilirubin level was 8 mg/dl. What is your impression?

A. pathologic jaundice
B. physiologic jaundice
C. neonatal sepsis
D. G6PD deficiency

A

B. physiologic jaundice

Rationale: According to Nelson’s Pediatrics, physiologic jaundice typically appears on the 2nd to 4th day of life and peaks around the 3rd to 5th day. A total serum bilirubin level of 8 mg/dl in the first 24-72 hours of life without other symptoms suggests physiologic jaundice, which is common in newborns due to increased bilirubin production and immature hepatic conjugation mechanisms.

How well did you know this?
1
Not at all
2
3
4
5
Perfectly
2
Q

A 1-month-old baby was brought to a pediatrician because of hypotonia and constipation. Birth history was uncomplicated. The mother also noted persistence of umbilical hernia and protrusion of the tongue. Which of the following metabolic disorders would you consider?

A. G6PD deficiency
B. Galactosemia
C. Congenital hypothyroidism
D. Hypoglycemia

A

C. Congenital hypothyroidism

Rationale: Nelson’s Pediatrics highlights that congenital hypothyroidism can present with symptoms such as hypotonia, constipation, prolonged jaundice, large fontanelles, umbilical hernia, and a large tongue. The persistence of an umbilical hernia and protrusion of the tongue are particularly characteristic.

How well did you know this?
1
Not at all
2
3
4
5
Perfectly
3
Q

The baby was delivered via C-section due to a non-reassuring fetal heart beat pattern. On PE, the baby was cyanotic, limp, in cardiorespiratory distress. You noted that the baby’s umbilical cord was stained with meconium. Few hours after birth, seizure was noted. Perinatal asphyxia was then considered. The following are the criteria for diagnosis of perinatal asphyxia EXCEPT:

A. Apgar < 3 beyond 5 minutes
B. CNS manifestations
C. Multiorgan failure
D. Blood pH 7.3

A

A. Apgar < 3 beyond 5 minutes

Rationale: Perinatal asphyxia is defined in Nelson’s Pediatrics as having at least one of the following: a 10-minute Apgar score of 5 or less, the need for resuscitation lasting more than 10 minutes, metabolic acidosis (pH ≤ 7.0) or base excess of -12 mmol/L in umbilical artery or within the first hour of life, and/or evidence of CNS involvement or multiorgan failure. An Apgar score of < 3 beyond 5 minutes indicates severe asphyxia, but for the diagnosis, the criteria are typically assessed within the first 10 minutes of life.

How well did you know this?
1
Not at all
2
3
4
5
Perfectly
4
Q

Polycythemia is a neonatal complication seen in:

A. Placenta previa
B. Abruptio placenta
C. Delayed clamping of the umbilical cord
D. Donor twin

A

C. Delayed clamping of the umbilical cord

Rationale: According to Nelson’s Pediatrics, delayed clamping of the umbilical cord can lead to an increased transfusion of blood from the placenta to the neonate, which can result in polycythemia (an abnormally high hematocrit).

How well did you know this?
1
Not at all
2
3
4
5
Perfectly
5
Q

Baby A was born full term via Cesarean section. He developed rapid respiration in the 3rd hour of his life. Chest x-ray showed widened intercostal spaces and fluid in interlobar fissures. The most likely diagnosis is:

A. Neonatal pneumonia
B. Transient tachypnea of the newborn
C. Hyaline membrane disease
D. Bronchopulmonary dysplasia

A

B. Transient tachypnea of the newborn

Rationale: Nelson’s Pediatrics describes transient tachypnea of the newborn (TTN) as a condition that commonly follows uneventful normal or cesarean deliveries. It presents with rapid breathing and characteristic x-ray findings, including prominent pulmonary vascular markings and fluid in the fissures. TTN usually resolves within 72 hours.

How well did you know this?
1
Not at all
2
3
4
5
Perfectly
6
Q

Term baby was delivered in respiratory distress with a note of drooling of saliva and difficulty in inserting orogastric tube during suctioning of secretions per orem. Chest x-ray showed coiling of the feeding tube in the upper esophagus. The infant is suspected to have:

A. Pyloric Stenosis
B. Intestinal Obstruction
C. Congenital diaphragmatic hernia
D. Tracheoesophageal fistula

A

D. Tracheoesophageal fistula

Rationale: The clinical presentation of drooling, respiratory distress, and inability to pass an orogastric tube, along with the x-ray finding of coiling in the upper esophagus, is characteristic of tracheoesophageal fistula, as described in Nelson’s Pediatrics.

How well did you know this?
1
Not at all
2
3
4
5
Perfectly
7
Q

A premature infant, 28 weeks by pediatric aging, developed abdominal distension on the 8th day of life with feeding intolerance and bloody stools. Which of the following is the most likely diagnosis?

A. Necrotizing Enterocolitis
B. Hypertrophic pyloric stenosis
C. Tracheoesophageal fistula
D. Congenital megacolon

A

A. Necrotizing Enterocolitis

Rationale: Nelson’s Pediatrics identifies necrotizing enterocolitis (NEC) as a serious gastrointestinal emergency in preterm infants. It presents with symptoms such as abdominal distension, feeding intolerance, and bloody stools, typically occurring around the first 2-3 weeks of life.

How well did you know this?
1
Not at all
2
3
4
5
Perfectly
8
Q

A radiographic finding that is pathognomonic of necrotizing enterocolitis (NEC) is:

A. Double bubble sign
B. Pneumatosis intestinalis
C. Absence of presacral gas
D. Thickened bowel loops

A

B. Pneumatosis intestinalis

Rationale: Nelson’s Pediatrics identifies pneumatosis intestinalis, which is the presence of gas within the wall of the intestines, as a pathognomonic radiographic finding for necrotizing enterocolitis (NEC). This finding confirms the clinical suspicion of NEC.

How well did you know this?
1
Not at all
2
3
4
5
Perfectly
9
Q

A preterm baby was born to an 18-year-old mother, birth weight is 900 g, and pediatric aging is equal to 30 weeks AOG. The baby started grunting and chest retractions in the 6th hour of life and was hooked up to a mechanical ventilator. What is the most likely diagnosis?

A. Hyaline membrane disease
B. Respiratory distress syndrome
C. Bronchopulmonary dysplasia
D. Transient tachypnea of the newborn

A

A. Hyaline Membrane disease

Rationale: Hyaline membrane disease, also known as respiratory distress syndrome (RDS), is common among preterm infants and is associated with surfactant deficiency. Nelson’s Pediatrics describes typical clinical features such as grunting, chest retractions, and the need for mechanical ventilation shortly after birth.

How well did you know this?
1
Not at all
2
3
4
5
Perfectly
10
Q

Radiographic findings of respiratory distress syndrome:

A. Fistuling of the lobes
B. Flattened diaphragm
C. Hyperinflated lung
D. Groundglass appearance

A

D. Groundglass appearance

Rationale: According to Nelson’s Pediatrics, the classic radiographic finding in respiratory distress syndrome (RDS) is a ground-glass appearance, which results from the diffuse atelectasis and decreased lung volumes due to surfactant deficiency.

How well did you know this?
1
Not at all
2
3
4
5
Perfectly
11
Q

A term baby weighing 4.3 kg is born without complications to a mother with gestational diabetes. At 12 hours of life, he appears mildly jaundiced. Vital signs are stable, he is feeding well and his blood type is the same as mother. Which of the following serum laboratory tests are most likely to help you evaluate this infant’s jaundice?

A. Total serum protein, serum albumin and liver transaminases
B. Total serum bilirubin and direct calcium level
C. Total serum bilirubin and glucose
D. Total serum bilirubin and hematocrit level

A

D. Total serum bilirubin and hematocrit level

Rationale: In Nelson’s Pediatrics, evaluating jaundice in newborns, especially those at risk for hemolytic disease, involves measuring total serum bilirubin to assess the severity of jaundice and hematocrit to check for signs of hemolysis or polycythemia.

How well did you know this?
1
Not at all
2
3
4
5
Perfectly
12
Q

In ABO blood group incompatibility:

A. Mother’s blood type A and baby’s blood type O
B. Mother’s blood type O and baby’s blood type A
C. Kernicterus is commonly seen
D. Jaundice occurs after 7 days of life

A

B. Mother’s blood type O and baby’s blood type A

Rationale: Nelson’s Pediatrics explains that ABO incompatibility occurs when a mother with blood type O has a baby with blood type A or B. This can lead to hemolytic disease of the newborn, which typically presents with jaundice within the first 24 hours of life, not after 7 days. Kernicterus is a severe complication but not commonly seen with ABO incompatibility due to early detection and treatment.

How well did you know this?
1
Not at all
2
3
4
5
Perfectly
13
Q

Baby Boy A, blood type B, Rh positive, was delivered full term with a good Apgar score. He developed jaundice in the 10th hour of life. His mother’s blood is Rh negative. What could have caused his jaundice?

A. Erythroblastosis fetalis
B. Vitamin K deficiency
C. Transplacental bleeding
D. Iron deficiency

A

A. Erythroblastosis fetalis

Rationale: Nelson’s Pediatrics explains that erythroblastosis fetalis occurs due to Rh incompatibility when an Rh-negative mother has an Rh-positive baby. This leads to hemolysis and jaundice within the first 24 hours of life.

How well did you know this?
1
Not at all
2
3
4
5
Perfectly
14
Q

Baby Boy A was born with a good Apgar score via uncomplicated repeat C-section. He was breastfed at the beginning of the 2nd hour of life. On the 5th hour, there was a note of vomiting of reddish mucus secretions. No associated symptoms noted. He passed a dark-colored stool on the 16th hour of life. Which of the following tests would tell that the baby’s condition is secondary to swallowed material?

A. CBC
B. Vitamin K assay
C. Kleihauer-Betke test
D. Apt test

A

D. Apt test

Rationale: According to Nelson’s Pediatrics, the Apt test differentiates fetal blood from maternal blood in the vomitus or stool. This test is used to determine if the bleeding is due to swallowed maternal blood.

How well did you know this?
1
Not at all
2
3
4
5
Perfectly
15
Q

Baby G was brought to the ER on the 2nd week of life because of yellowish discoloration of skin. He was lethargic with opisthotonus and spasticity. Total bilirubin was 30 mg/dl. The most likely diagnosis is:

A. Sepsis neonatorum
B. Perinatal asphyxia
C. Bilirubin encephalopathy
D. Persistent Pulmonary Hypertension

A

C. Bilirubin encephalopathy

Rationale: Nelson’s Pediatrics states that bilirubin encephalopathy (kernicterus) is caused by high levels of unconjugated bilirubin (>20 mg/dl), leading to central nervous system manifestations such as lethargy, opisthotonus, and spasticity.

How well did you know this?
1
Not at all
2
3
4
5
Perfectly
16
Q

A morbidity that is associated with monochorionic twinning involves the possibility of twin-to-twin transfusion syndrome (TTTS):

A. Growth retardation in the recipient twin
B. Growth retardation in the donor twin
C. Polycythemia in the donor twin
D. Anemia in the recipient twin

A

B. Growth retardation in the donor twin

Rationale: Nelson’s Pediatrics explains that in twin-to-twin transfusion syndrome (TTTS), the donor twin experiences growth retardation and anemia due to decreased blood flow, while the recipient twin may develop polycythemia and hypervolemia.

How well did you know this?
1
Not at all
2
3
4
5
Perfectly
17
Q

What is the most common cause of neonatal unconjugated hyperbilirubinemia?

A. Hemolytic disorders
B. Bacterial sepsis
C. Extrahepatic obstruction
D. Genetic disorder (e.g., Rotor-Dubin Johnson)

A

B. Bacterial sepsis

How well did you know this?
1
Not at all
2
3
4
5
Perfectly
18
Q

Neonatal jaundice is pathologic in all of the following conditions EXCEPT:

A. Clinical jaundice in the first 24 hours of life
B. If total serum bilirubin increased by 5 mg/dL/day
C. If direct serum bilirubin or B2 exceeds 1.5-2 mg/dL (> 10% of total serum bilirubin)
D. If clinical jaundice appears at 1 week or less in full term infant or 2 weeks or less in preterm infants

A

D. If clinical jaundice appears at 1 week or less in full term infant or 2 weeks or less in preterm infants

Rationale: Nelson’s Pediatrics indicates that jaundice appearing within the first week of life in full-term infants or within the first two weeks in preterm infants is generally physiologic. Pathologic jaundice presents earlier (within 24 hours) or persists longer than these time frames.

How well did you know this?
1
Not at all
2
3
4
5
Perfectly
19
Q

The neonate may acquire infection through the placenta. This organism is rarely transmitted through the placenta:
A. Toxoplasmosis
B. Herpes simplex
C. Cytomegalovirus
D. Rubella

A

Herpes simplex
Rationale: Nelson’s Pediatrics notes that herpes simplex virus is rarely transmitted through the placenta compared to other organisms like Toxoplasmosis and Rubella.

How well did you know this?
1
Not at all
2
3
4
5
Perfectly
20
Q

ABO incompatibility can be differentiated from Rh incompatibility in that the former is:
A. Milder, hydrops is not common
B. Mother is Rh positive, baby is Rh negative
C. Weakly positive on Coomb’s test
D. Occurs during the 2nd pregnancy

A

milder, hydrops not common
Rationale: Nelson’s Pediatrics describes ABO incompatibility as generally milder than Rh incompatibility, with less risk of severe outcomes such as hydrops fetalis.

How well did you know this?
1
Not at all
2
3
4
5
Perfectly
21
Q

Which of the following decreases the risk of neurologic damage in a jaundiced newborn?
A. Maternal ingestion of phenobarbital during pregnancy
B. Hypoalbuminemia
C. Acidosis
D. Sepsis

A

Maternal ingestion of phenobarbital during pregnancy
Rationale: According to Nelson’s Pediatrics, maternal ingestion of phenobarbital can induce hepatic glucuronyl transferase in the fetus, decreasing the risk of bilirubin-induced neurologic damage.

How well did you know this?
1
Not at all
2
3
4
5
Perfectly
22
Q

A baby was born small for gestational age (SGA) with opacities of both eyes and a grade 3/6 machinery-like murmur. The most likely diagnosis is:
A. Congenital toxoplasmosis
B. Congenital rubella syndrome
C. Congenital herpes infection
D. Congenital syphilis

A

Congenital rubella syndrome
Rationale: Nelson’s Pediatrics states that congenital rubella syndrome commonly presents with features such as cataracts, congenital heart disease, and microcephaly, which match the clinical description provided.

How well did you know this?
1
Not at all
2
3
4
5
Perfectly
23
Q

Which of the following statements regarding late-onset sepsis is NOT correct?
A. It is common after one week of life until 60 days
B. It is either community or hospital acquired infection
C. Meningitis and necrotizing enterocolitis are common complications
D. It is usually caused by E. coli infection

A

It is usually caused by E. coli infection
Rationale: Nelson’s Pediatrics indicates that late-onset sepsis in neonates is more commonly caused by organisms like Group B Streptococcus (GBS) rather than E. coli, which is more associated with early-onset sepsis.

How well did you know this?
1
Not at all
2
3
4
5
Perfectly
24
Q

The most common etiologic agent for early-onset sepsis is:
A. Staphylococcus aureus
B. Listeria monocytogenes
C. Group B Streptococcus (GBS)
D. Streptococcus pneumoniae

A

Group B Streptococcus (GBS)
Rationale: Nelson’s Pediatrics identifies Group B Streptococcus (GBS) as the most common etiologic agent for early-onset neonatal sepsis, usually presenting within the first week of life.

How well did you know this?
1
Not at all
2
3
4
5
Perfectly
25
Q

Baby Girl A was delivered full term at 40 weeks, weighing 1.5 kg, with chorioretinitis, hepatomegaly, thrombocytopenia, cerebral calcification, and hydrocephalus. What is the most likely diagnosis?
A. Congenital toxoplasmosis
B. Congenital hepatitis B
C. Congenital herpes simplex
D. Congenital rubella

A

. Congenital toxoplasmosis
Rationale: Nelson’s Pediatrics outlines that congenital toxoplasmosis can present with findings such as hydrocephalus, intracranial calcifications, and chorioretinitis, consistent with the clinical features described.

How well did you know this?
1
Not at all
2
3
4
5
Perfectly
26
Q

A newborn is examined for a gastrointestinal problem. An upper GI series shows abnormal positioning of the ligament of Treitz and the cecum. Which of the following is most likely?
A. Gastroesophageal reflux
B. Pyloric stenosis
C. Hirschsprung disease
D. GI malrotation

A

GI malrotation
Rationale: Nelson’s Pediatrics explains that malrotation involves abnormal positioning of the intestines due to incomplete rotation during fetal development, which is confirmed by upper GI series showing abnormal positioning of the ligament of Treitz and the cecum.

How well did you know this?
1
Not at all
2
3
4
5
Perfectly
27
Q

A 1-day-old infant born by a difficult forceps delivery is alert and active but does not move her left arm, which is kept internally rotated by her side with the forearm extended and pronated. Moro reflex is absent on the left arm. The rest of her physical examination is normal. This clinical picture most likely indicates:
A. Fracture of the left clavicle
B. Fracture of the left humerus
C. Erb’s palsy on the left arm
D. Phrenic nerve paralysis

A

Erb’s palsy on the left arm
Rationale: According to Nelson’s Pediatrics, Erb’s palsy, caused by injury to the brachial plexus during delivery, presents with the characteristic “waiter’s tip” posture of the affected arm, consistent with the clinical picture described.

How well did you know this?
1
Not at all
2
3
4
5
Perfectly
28
Q

The best way to prevent the complications associated with prematurity is:
A. Administration of antenatal steroids
B. Adequate nutrition
C. Maternal multivitamin supplement
D. Aggressive neonatal resuscitation

A

Administration of antenatal steroids
Rationale: Nelson’s Pediatrics emphasizes that antenatal steroids are the most effective intervention to prevent complications of prematurity, such as respiratory distress syndrome, by accelerating fetal lung maturity.

How well did you know this?
1
Not at all
2
3
4
5
Perfectly
29
Q

Which of the following is suggestive of congenital infection?
A. Increasing IgM titer
B. Decreasing IgM titer
C. Increasing IgG titer
D. Decreasing IgG titer

A

Increasing IgM titer
Rationale: Nelson’s Pediatrics states that an increasing IgM titer in the fetus or newborn is indicative of a congenital infection because IgM does not cross the placenta, and its presence suggests in utero infection.

How well did you know this?
1
Not at all
2
3
4
5
Perfectly
30
Q

Baby Boy M was born full term. Labor was uneventful. He was given breast milk. He was apparently well when he was noted to have yellowish discoloration of the skin on the 30th hour of life. No associated symptoms were noted. Total serum bilirubin level was 8 mg/dl. What is your impression?
A. Pathologic jaundice
B. Physiologic jaundice
C. Neonatal sepsis
D. G6PD deficiency

A

physiologic jaundice
Rationale: Nelson’s Pediatrics describes physiologic jaundice as typically appearing on the second or third day of life and is characterized by a bilirubin level that rises less than 5 mg/dL per day and peaks at less than 12 mg/dL in term infants.

How well did you know this?
1
Not at all
2
3
4
5
Perfectly
31
Q

A newborn baby was noted to have yellowish discoloration of the skin on the 24th hour of life. Complete blood count showed low hemoglobin and hematocrit. Total bilirubin was 21 mg/dL. The appropriate initial treatment option would be:
A. Exposure to phototherapy
B. Do septic work-up
C. Do exchange transfusion immediately
D. Administration of IV immunoglobulin

A

Exposure to phototherapy
Rationale: According to Nelson’s Pediatrics, phototherapy is the initial treatment for neonatal hyperbilirubinemia to lower bilirubin levels and prevent bilirubin-induced neurologic dysfunction.

How well did you know this?
1
Not at all
2
3
4
5
Perfectly
32
Q

Baby JM showed deepening jaundice after phototherapy started, and repeat bilirubin determination increased to 30 mg/dL. What will be the next treatment plan for this baby?
A. Continue exposure to phototherapy
B. Double volume exchange transfusion
C. Metalloporphyrin
D. Phenobarbital

A

Double volume exchange transfusion
Rationale: Nelson’s Pediatrics recommends double volume exchange transfusion when phototherapy fails, and bilirubin levels rise dangerously high, to rapidly reduce bilirubin levels and prevent kernicterus.

How well did you know this?
1
Not at all
2
3
4
5
Perfectly
33
Q

Maternal drug addiction commonly results in:
A. Normal birth weight
B. Neonatal drug withdrawal
C. LGA (large for gestational age)
D. No significant effect on the fetus

A

Neonatal drug withdrawal
Rationale: Nelson’s Pediatrics explains that maternal drug addiction, particularly to opioids, commonly results in neonatal drug withdrawal syndrome, characterized by a range of symptoms including irritability, poor feeding, and seizures.

How well did you know this?
1
Not at all
2
3
4
5
Perfectly
34
Q

Neonatal thyrotoxicosis is common among mothers with:
A. Hypothyroidism
B. Diabetes Mellitus
C. Grave’s Disease

A

Grave’s Disease
Rationale: Nelson’s Pediatrics identifies maternal Graves’ disease as a common cause of neonatal thyrotoxicosis due to the transfer of thyroid-stimulating antibodies from the mother to the fetus.

How well did you know this?
1
Not at all
2
3
4
5
Perfectly
35
Q

The likelihood of having a baby with hyaline membrane disease (RDS) is reduced in pregnancies associated with:
A. C-section delivery
B. Diabetes Mellitus
C. Chronic Hypertension
D. Premature Labor

A

Chronic Hypertension
Rationale: Nelson’s Pediatrics does not list chronic hypertension as a protective factor against hyaline membrane disease. Instead, conditions like diabetes mellitus, premature labor, and cesarean section delivery are associated with a higher risk.

How well did you know this?
1
Not at all
2
3
4
5
Perfectly
36
Q

Which of the following statements about congenital duodenal obstruction is true?
A. Abdominal x-ray will show “double-bubble” sign
B. Abdominal distension is more prominent
C. Non-bilious vomiting
D. Sawtooth contractions of the aganglionic segment

A

Abdominal x-ray will show “double-bubble” sign
Rationale: Nelson’s Pediatrics describes the “double-bubble” sign on an abdominal x-ray as a classic indication of congenital duodenal obstruction, showing air in the stomach and the proximal duodenum.

How well did you know this?
1
Not at all
2
3
4
5
Perfectly
37
Q

A 3-day-old baby was brought to the emergency room because of stooling since birth. This was associated with bilious vomiting and abdominal distension. All of the following laboratory exams should be done EXCEPT:
A. Plain abdomen
B. Barium enema
C. Gastroscopy
D. Rectal biopsy

A

Gastroscopy
Rationale: Nelson’s Pediatrics mentions that while gastroscopy can be used for various indications, it is not typically performed in the acute evaluation of neonatal bowel obstruction where plain abdominal x-ray, barium enema, or rectal biopsy are more commonly used.

How well did you know this?
1
Not at all
2
3
4
5
Perfectly
38
Q

A 4-day-old newborn baby was noted to have not passed meconium since birth. There was abdominal distension, and the baby was referred to a pediatric surgeon. Hirschsprung disease was considered. What is the pathology in this condition?
A. Absence of ganglion cells in the bowel
B. Extensive bowel necrosis
C. Incomplete rotation of the cecum
D. Hypertrophy of the circular muscles of the pylorus

A

Absence of ganglion cells in the bowel
Rationale: Nelson’s Pediatrics describes Hirschsprung disease as a congenital disorder characterized by the absence of ganglion cells in the submucosal and myenteric plexuses of the bowel. This results in a functional obstruction, leading to symptoms such as failure to pass meconium, abdominal distension, and constipation.

How well did you know this?
1
Not at all
2
3
4
5
Perfectly
39
Q

Prenatal factors associated with increased risk of later childhood obesity include all except:
A. Low birth weight
B. Large for gestational age
C. High preconceptual weight
D. Maternal smoking

A

Maternal smoking
Rationale: Nelson’s Pediatrics notes that low birth weight, large for gestational age, and high preconceptual weight are all associated with an increased risk of childhood obesity. However, maternal smoking is typically associated with low birth weight and does not contribute to childhood obesity.

How well did you know this?
1
Not at all
2
3
4
5
Perfectly
40
Q

Toilet training, which occurs in the toddler and preschool years, is usually completed at what age?
A. 5 y/o
B. 4 y/o
C. 6 y/o
D. 7 y/o

A

4 y/o
Rationale: According to Nelson’s Pediatrics, toilet training is usually completed by the age of 4 years. Most children achieve bladder and bowel control between 18 and 24 months, with some children taking longer.

How well did you know this?
1
Not at all
2
3
4
5
Perfectly
41
Q

Enuresis is repeated voiding of urine in clothes or beds at least 2 times/week for at least 3 consecutive months in a child who is at least:
A. 5 y/o
B. 4 y/o
C. 6 y/o
D. 7 y/o

A

5 y/o
Rationale: Nelson’s Pediatrics defines enuresis as repeated voiding of urine into bed or clothes at least twice a week for three consecutive months in a child who is at least 5 years old.

How well did you know this?
1
Not at all
2
3
4
5
Perfectly
42
Q

Children with secondary nocturnal enuresis may have:
A. UTI
B. Diabetes mellitus
C. Diabetes insipidus
D. Psychosocial stressor

A

Psychosocial stressor
Rationale: Nelson’s Pediatrics states that secondary nocturnal enuresis, which is the reappearance of bedwetting after a period of dryness, can be associated with psychosocial stressors such as family disruptions, school problems, or other stress-inducing events.

How well did you know this?
1
Not at all
2
3
4
5
Perfectly
43
Q

Encopresis is the regular, voluntary or involuntary passage of feces into a place other than the toilet after the age of:
A. 2 yrs
B. 3 yrs
C. 4 yrs
D. 5 yrs

A

4 yrs
Rationale: Nelson’s Pediatrics defines encopresis as the passage of feces into inappropriate places, such as clothing or the floor, after the age of 4 years. This is the age by which most children have typically achieved bowel control.

How well did you know this?
1
Not at all
2
3
4
5
Perfectly
44
Q

A 15-month-old infant is referred for speech-language evaluation if she:
A. Does not follow simple instructions
B. Does not point to pictures
C. Does not look or point at 5-10 objects
D. Does not say “mama,” “dada,” or other names

A

does not look or point at 5-10 objects
Rationale: Nelson’s Pediatrics indicates that by 15 months, a child should be able to look at or point to 5-10 objects when named. Failure to meet this milestone may warrant a speech and language evaluation.

How well did you know this?
1
Not at all
2
3
4
5
Perfectly
45
Q

A 2-month-old infant presented with daily paroxysms of continuous crying for > 3 hours starting after 6-8 pm, associated with facial grimacing, leg flexion, and passing flatus. On examination, no abnormality was detected apart from mild abdominal distension. Of the following, the MOST likely cause is:
A. Hunger
B. Wet diaper
C. Overfeeding
D. Colic

A

Colic
Rationale: Nelson’s Pediatrics defines colic as episodes of crying for more than three hours a day, more than three days a week, for three weeks or longer in an otherwise healthy infant. This typically starts around 6-8 weeks of age and is characterized by paroxysms of fussiness and crying, often in the evening.

How well did you know this?
1
Not at all
2
3
4
5
Perfectly
46
Q

After full evaluation of an infant with excessive crying due to colic, an organic etiology was found in less than:
A. 5% of infants
B. 10% of infants
C. 15% of infants
D. 20% of infants

A

5% of infants
Rationale: According to Nelson’s Pediatrics, organic etiologies for excessive crying are found in less than 5% of infants after thorough evaluation. Most cases of colic are functional and not due to underlying organic disease.

How well did you know this?
1
Not at all
2
3
4
5
Perfectly
47
Q

When is the best age for a physician to administer a developmental screening tool in children?
A. 1 year
B. 2 years
C. 6 years
D. 10 years

A

2 years
Rationale: Nelson’s Pediatrics recommends developmental screening at 9, 18, and 24 or 30 months of age to identify any developmental delays early. The age of 2 years is a critical time for screening to catch developmental issues before they become more pronounced.

How well did you know this?
1
Not at all
2
3
4
5
Perfectly
48
Q

A 3-year-old child has recurrent attacks of screaming, vomiting, and biting other children. These attacks last 2-5 minutes and occur once or twice weekly; the child looks well between the attacks. Of the following, the MOST likely cause is:
A. Autism
B. Traumatic brain injury
C. Cognitive impairment
D. Temper tantrums

A

temper tantrums
Rationale: Nelson’s Pediatrics states that temper tantrums are common in toddlers and preschool-aged children. They are typically normal expressions of frustration or unmet needs and do not usually indicate a serious underlying condition when occurring in an otherwise well child.

How well did you know this?
1
Not at all
2
3
4
5
Perfectly
49
Q

In the DSM-5 criteria of ADHD, which of the following statements is true?
A. The behavior must be developmentally inappropriate
B. It must begin before 7 years of age
C. It must be present for at least 3 months
D. It must be present in 2 or more settings and reported as such by independent observers

A

It must be present in 2 or more settings and reported as such by independent observers
Rationale: According to Nelson’s Pediatrics, the DSM-5 criteria for ADHD require that symptoms be present in two or more settings (e.g., at home and school) and that they be observed and reported by others to ensure the behavior is consistent across different environments.

How well did you know this?
1
Not at all
2
3
4
5
Perfectly
50
Q

Diagnosis of ADHD in children up to the age of 16 years requires the presence of at least either or both:
A. 4 of 9 inattentive symptoms or 4 of 9 hyperactive-impulsive symptoms for at least 4 months in two or more environments
B. 5 of 9 inattentive symptoms or 5 of 9 hyperactive-impulsive symptoms for at least 5 months in two or more environments
C. 6 of 9 inattentive symptoms or 6 of 9 hyperactive-impulsive symptoms for at least 6 months in two or more environments
D. 6 of 9 inattentive symptoms or 6 of 9 hyperactive-impulsive symptoms for at least 3 months

A

C. 6 of 9 inattentive symptoms or 6 of 9 hyperactive-impulsive symptoms for at least 6 months in two or more environmentsRationale: Nelson’s Pediatrics specifies that a diagnosis of ADHD in children up to age 16 requires the presence of at least 6 out of 9 inattentive symptoms or 6 out of 9 hyperactive-impulsive symptoms for at least six months in two or more settings.

How well did you know this?
1
Not at all
2
3
4
5
Perfectly
51
Q

Diagnosis of autistic spectrum disorder (ASD) depends partly but importantly on assessment of language. The following may raise concern regarding language development and may indicate ASD, except:
A. Absent babbling by 6 months
B. Absent gestures by 1 year
C. Absent 2-word purposeful phrases by 2 years
D. Loss of language or social skills at any time

A

Absent babbling by 6 months
Rationale: Nelson’s Pediatrics states that concerns for ASD typically arise if there is absent babbling by 12 months, absent gestures by 12 months, no two-word spontaneous phrases by 24 months, or any loss of language or social skills at any age. Babbling by 6 months is not a diagnostic criterion.

How well did you know this?
1
Not at all
2
3
4
5
Perfectly
52
Q

Preschoolers are said to be in this stage of psychosocial development:
A. Autonomy vs Shame
B. Initiative vs Guilt
C. Trust vs Mistrust
D. Industry vs Inferiority

A

Initiative vs Guilt
Rationale: According to Nelson’s Pediatrics and Erikson’s stages of psychosocial development, preschool-aged children (3-5 years) are in the “Initiative vs Guilt” stage, where they begin to assert power and control over their environment through directing play and other social interactions.

How well did you know this?
1
Not at all
2
3
4
5
Perfectly
53
Q

A 3-year-old boy was brought by his parents because they think he is deaf. He shows no interest in them or anyone around him, speaks only in broken sentences, and often lines his toys in a straight line. The hearing test is normal. The most likely diagnosis is:
A. ADHD
B. Dyslexia
C. Autism Spectrum Disorder
D. Aphasia

A

Autism Spectrum Disorder
Rationale: Nelson’s Pediatrics outlines that children with autism spectrum disorder (ASD) may exhibit behaviors such as lining up toys, a lack of interest in others, and speech delays. These behaviors are consistent with the description provided.

How well did you know this?
1
Not at all
2
3
4
5
Perfectly
54
Q

Refers to difficulty in developing an ideomotor plan and activating coordinated and integrated visual-motor actions to complete a task or solve a motor problem such as assembling a model:
A. Dyslexia
B. Dyspraxia
C. Aphasia
D. Astereognosis

A

Dyspraxia
Rationale: Nelson’s Pediatrics describes dyspraxia as a disorder that affects motor planning and coordination, making it difficult for children to perform tasks requiring fine and gross motor skills, such as assembling a model.

How well did you know this?
1
Not at all
2
3
4
5
Perfectly
55
Q

If a child’s speech is not understood by age 3 to 4 months, the concern could be:
A. Familial disorders
B. Hearing impairment
C. Speech and language delay
D. Developmental disorders

A

Familial disorders
Rationale: Nelson’s Pediatrics states that speech delays between 3 to 4 months can often be related to familial factors or genetic predispositions, among other causes.

How well did you know this?
1
Not at all
2
3
4
5
Perfectly
56
Q

The most common of the learning disabilities affecting 80% of children is:
A. Dyslexia
B. Dysgraphia
C. Dyscalculia
D. Dyspraxia

A

Dyslexia
Rationale: Nelson’s Pediatrics notes that dyslexia is the most common learning disability, affecting approximately 80% of children with learning disabilities. It primarily impacts reading and language skills.

How well did you know this?
1
Not at all
2
3
4
5
Perfectly
57
Q

Regarding “breath holding spells” in children, one of the following statements is NOT true:
A. It is fairly common in the first years of life
B. It does not contribute to an increased risk of seizure disorder
C. Parents are advised to ignore and not to reinforce these attacks
D. It must be immediately attended to prevent hypoxia and onset of seizures

A

It must be immediately attended to prevent hypoxia and onset of seizures
Rationale: Nelson’s Pediatrics explains that while breath-holding spells are usually benign and self-limited, immediate attention is required if the spell is prolonged to prevent hypoxia and the risk of seizures.

How well did you know this?
1
Not at all
2
3
4
5
Perfectly
58
Q

The mother of a healthy 2-year-old girl is concerned about her daughter’s thumb sucking behavior. The mother is worried that it may continue or cause dental problems. Which of the following information is most appropriate to give the mother?
A. Reassurance to the mother
B. Leave the behavior as complications usually start after 5 years
C. Ignore thumb sucking and encourage a substituted behavior
D. Use of bitter ointments will resolve the problem

A

ignore thumb sucking and encouraging a substituted behavior
Rationale: Nelson’s Pediatrics suggests that thumb sucking is a normal behavior in infancy and early childhood. Encouraging parents to ignore the behavior and provide positive reinforcement for substitute behaviors is often effective.

How well did you know this?
1
Not at all
2
3
4
5
Perfectly
59
Q

Some of the common risk behaviors in adolescents include the use of alcohol, tobacco, and drugs. In the country, the typical order of prevalence is:
A. Drug users > Smokers > Alcohol drinkers
B. Smokers > Alcohol drinkers > Drug users
C. Alcohol drinkers > Smokers > Drug users
D. Smokers > Drug users > Alcohol drinkers

A

Alcohol drinkers > smokers > drug users
Rationale: According to Nelson’s Pediatrics, the most common risk behaviors among adolescents include alcohol use, followed by tobacco use, and then drug use.

How well did you know this?
1
Not at all
2
3
4
5
Perfectly
60
Q

A standard drink contains around 14 grams of alcohol. Which of the following contains the least percentage of alcohol in terms of alcohol/volume?
A. Gin
B. Vodka
C. Wine
D. Malt liquor

A

Malt liquor
Rationale: Nelson’s Pediatrics indicates that malt liquor typically contains a lower percentage of alcohol by volume compared to spirits like gin, vodka, and wine.

How well did you know this?
1
Not at all
2
3
4
5
Perfectly
61
Q

There was a rave party in one of the bars near your place. Different alcohols were served, and teens began binge drinking. They served all the drinks in a 12oz glass. Which one of the following is most at risk of having an alcohol overdose?
A. A 14-year-old girl who drank 3 glasses of beer in 3 hours
B. A 15-year-old boy who drank 2 glasses of wine in 2 hours
C. A 16-year-old girl who drank 1 glass of rum in 2 hours
D. A 17-year-old boy who drank 4 glasses of malt liquor in 3 hours

A

C. A 16-year-old girl who drank 1 glass of rum in 2 hours
Rationale: According to Nelson’s Pediatrics, binge drinking is defined as consuming a substantial amount of alcohol in a short period, typically within 2 hours. A standard drink of distilled spirits like rum contains a higher concentration of alcohol compared to beer or wine. Although the volume of a single drink may seem smaller, the alcohol content is much higher, making it more potent and risky. A single 12 oz glass of rum is far above the standard drink size for spirits (typically 1.5 oz per drink), significantly increasing the risk of alcohol overdose, especially in a young person with potentially lower tolerance. Thus, a 16-year-old girl consuming such a high alcohol content in a short period places her at a considerable risk of alcohol overdose

How well did you know this?
1
Not at all
2
3
4
5
Perfectly
62
Q

The most commonly abused substance among adolescents is:
A. Methamphetamine
B. Marijuana
C. Rugby Inhalant
D. Alcohol

A

Alcohol
Rationale: Nelson’s Pediatrics highlights that alcohol is the most commonly abused substance among adolescents, surpassing the use of other substances like methamphetamine, marijuana, and inhalants.

How well did you know this?
1
Not at all
2
3
4
5
Perfectly
63
Q

Which of the following statements regarding violence is true?
A. Cyberbullying is very common but is never disclosed or reported among the youth
B. Media decreases the risk of violence among the youth
C. There are more youth who admit to hurting someone than being hurt by someone in the past 12 months
D. Family and the environment are the only factors to consider in screening for violence

A

There are more youth who admits to hurting someone than being hurt by someone in the past 12 months
Rationale: Nelson’s Pediatrics reports that a higher percentage of youth admit to aggressive behaviors such as hurting someone compared to those who report being victims of violence.

64
Q

Which of the following factors are impacts of early teen pregnancy on the fetus and newborn?
A. Low birth weight, Malnutrition, Pre-eclampsia
B. Fetal mortality, Low birth weight, Malnutrition
C. Malnutrition, Fetal mortality, Neonatal pneumonia
D. Perinatal asphyxia, LGA

A

Fetal mortality, Low birth weight, Malnutrition
Rationale: Nelson’s Pediatrics identifies that early teen pregnancy impacts the fetus and newborn through increased risks of fetal mortality, low birth weight, and malnutrition, along with other complications.

65
Q

In dealing with adolescents who present with risk-taking behavior, we must:
A. Ask the adolescents together with their parents regarding their risk-taking behavior
B. Do a complete sexual history even if they only came in with a chief complaint of cough
C. Disclose to the parents that their teen tried drinking a glass of beer for the first time in the last party they were allowed to attend
D. Skip the complete physical examination and just do a laboratory test if they admit to doing drugs

A

Do a complete sexual history even if they only came in with a chief complaint of cough
Rationale: Nelson’s Pediatrics recommends taking a complete sexual history as part of the HEADSS assessment (Home, Education/Employment, Activities, Drugs, Sexuality, Suicide/Depression) to identify risk-taking behaviors in adolescents, regardless of their presenting complaint.

66
Q

A 12-year-old came with a chief complaint of irregular menstruation, having it every 2-3 months. She said she had her menarche 10 months ago and she does not have any other complaints. Your physical examination was normal. Her irregular menstruation is due to:
A. Anovulation
B. Pregnancy
C. Polycystic ovarian syndrome
D. Mass/Malignancy

A

Anovulation
Rationale: Nelson’s Pediatrics explains that irregular menstruation within the first two years after menarche is commonly due to anovulation. This is a normal part of the maturation process of the hypothalamic-pituitary-ovarian axis.

67
Q

Primary amenorrhea is the lack of menses by what age in the absence of secondary sexual characteristics?
A. 13
B. 14
C. 15
D. 16

A

A. 13 years old in the absence of secondary sexual characteristics.

68
Q

A patient presents with mucopurulent penile discharge after a one-night stand. Physical examination shows normal findings. What is the most likely diagnosis?
A. Gonorrhea
B. Chlamydia
C. Herpes
D. Syphilis

A

Chlamydia
Rationale: Nelson’s Pediatrics notes that chlamydia is a common cause of mucopurulent cervicitis and urethritis in adolescents. It often presents with mucopurulent discharge and can be asymptomatic in many cases.

69
Q

A morbidity that is associated with monochorionic twinning involves the possibility of twin-to-twin transfusion syndrome (TTTS):

a. Growth retardation in the recipient twin
b. Growth retardation in the donor twin
c. Polycythemia in the donor twin
d. Anemia in the recipient twin

A

b. Growth retardation in the donor twin

Rationale: Twin-to-twin transfusion syndrome (TTTS) occurs in monochorionic twin pregnancies where there is an imbalance in blood flow between the twins through vascular anastomoses in the shared placenta. The donor twin transfers blood to the recipient twin, leading to growth retardation and anemia in the donor twin (option b). The recipient twin, on the other hand, may experience polycythemia, hypervolemia, and potentially heart failure, not growth retardation (option a) or anemia (option d). Polycythemia in the donor twin (option c) is incorrect, as it is typically seen in the recipient twin.

70
Q

A 6-day-old infant was seen in the ER for fever and poor feeding. The baby was delivered vaginally at 36 weeks gestation with a birth weight of 2900 grams. Maternal laboratory tests were negative. The most likely organism causing the baby’s symptoms is:

a. Staphylococcus pneumonia
b. Group B Streptococcus
c. Haemophilus influenzae
d. Listeria monocytogenes

A

b. Group B Streptococcus

Rationale: Group B Streptococcus (GBS) is a common cause of neonatal sepsis, particularly in early-onset disease (within the first week of life). Despite negative maternal laboratory tests, GBS can still be transmitted during vaginal delivery. Staphylococcus pneumonia (option a) and Haemophilus influenzae (option c) are less common causes in this age group, while Listeria monocytogenes (option d) can cause neonatal infection but is less common than GBS.

71
Q

A full-term newborn is delivered vaginally. At 1 minute of life, the newborn’s respiratory rate is slow and irregular with a heart rate of 80 beats/minute. There is some flexion of his upper and lower extremities, and he does not respond when a catheter is placed into his nose. He is cyanotic and pale. What is the infant’s 1-minute Apgar score?

a. 2
b. 3
c. 4
d. 5

A

b. 3

Rationale: The Apgar score at 1 minute is calculated as follows:

Appearance (Color): 0 (cyanotic and pale)
Pulse (Heart rate): 1 (heart rate of 80)
Grimace (Reflex irritability): 0 (no response to nasal catheter)
Activity (Muscle tone): 1 (some flexion)
Respiration: 1 (slow and irregular)
Total: 0 + 1 + 0 + 1 + 1 = 3

72
Q

A 34-year-old mother delivered a full-term baby with a birth weight of 3 kg. APGAR scores were 7 and 9 at 1 and 5 minutes, respectively. At birth, the baby developed chest retraction and cyanosis. Suctioning of secretions was done. Whenever the baby cries and opens his mouth, he turns pink. What is the most likely diagnosis?

a. Choanal atresia
b. Esophageal atresia
c. Duodenal atresia
d. Hypoplasia of the mandible

A

a. Choanal atresia

Rationale: Choanal atresia is a congenital condition where the nasal passages are blocked, leading to cyanosis that improves when the baby cries (as breathing occurs through the mouth). Esophageal atresia (option b), duodenal atresia (option c), and hypoplasia of the mandible (option d) do not typically present with this specific pattern of cyanosis and improvement upon crying.

73
Q

A newborn baby was noted to be cyanotic and limp at birth. He was also noted to have difficulty in respiration. On physical examination, the following findings were seen: the abdomen was scaphoid, increased AP diameter of the chest, displaced point of maximum impulse, and breath sounds absent on the left lung. The most likely diagnosis is:

a. Diaphragmatic hernia
b. Pneumothorax
c. Pneumonia
d. Transient tachypnea of the newborn

A

a. Diaphragmatic hernia

Rationale: A diaphragmatic hernia is characterized by herniation of abdominal contents into the chest cavity, leading to respiratory distress, a scaphoid abdomen, and absent breath sounds on the affected side. Pneumothorax (option b), pneumonia (option c), and transient tachypnea of the newborn (option d) do not typically present with these findings.

74
Q

At 43 weeks gestation, the infant is delivered apneic, limp, pale, and covered with “pea soup” thickly meconium-stained amniotic fluid. The initial step in the resuscitation of this infant at delivery should be:

a. Artificial ventilation with bag and mask
b. Endotracheal intubation to suction meconium
c. Give oxygen by mask
d. Catheterization of the umbilical vein

A

b. Endotracheal intubation to suction meconium

Rationale: For infants born through meconium-stained amniotic fluid who are not vigorous (apneic, limp, pale), the initial step is to perform endotracheal intubation to suction meconium from the airway to prevent aspiration and further complications. Artificial ventilation with bag and mask (option a) and giving oxygen by mask (option c) are important but should follow initial suctioning. Catheterization of the umbilical vein (option d) is not the first step.

75
Q

The most common cause of sudden cardio-respiratory deterioration in a newborn patient who is receiving positive pressure ventilation is:

a. Bronchopulmonary dysplasia
b. Reopening of the ductus arteriosus
c. Tension pneumothorax
d. Sepsis neonatorum

A

c. Tension pneumothorax

Rationale: Tension pneumothorax is a common and potentially life-threatening cause of sudden cardio-respiratory deterioration in a newborn receiving positive pressure ventilation. It occurs due to air trapping in the pleural space, causing lung collapse and mediastinal shift. Bronchopulmonary dysplasia (option a) and reopening of the ductus arteriosus (option b) are less common and not typically sudden causes. Sepsis neonatorum (option d) can cause deterioration but not as acutely as a tension pneumothorax.

76
Q

Aspiration of meconium-stained amniotic fluid is usually associated with:

a. Congenital heart disease
b. Congenital megacolon
c. Tracheoesophageal fistula
d. Perinatal asphyxia

A

d. Perinatal asphyxia

Rationale: Aspiration of meconium-stained amniotic fluid is associated with perinatal asphyxia, where the fetus experiences hypoxia and may pass meconium into the amniotic fluid. Congenital heart disease (option a), congenital megacolon (option b), and tracheoesophageal fistula (option c) are not typically associated with meconium aspiration.

77
Q

A newborn baby was lethargic and bradycardic (HR 40/min). Oxygen was administered via bag and mask, and he was intubated. His HR remained at 40 beats/min. Which of the following is the most appropriate next step?

a. Administer epinephrine
b. Administer sodium bicarbonate
c. Administer atropine
d. Perform chest compressions

A

d. Perform chest compressions

Rationale: If the heart rate remains below 60 beats per minute after adequate ventilation (including intubation), the next step is to initiate chest compressions to improve cardiac output. Administering epinephrine (option a) may be considered if the heart rate does not improve with compressions. Sodium bicarbonate (option b) and atropine (option c) are not the immediate next steps.

78
Q

A decreasing cardiac rate (<60 per minute) after providing positive pressure ventilation is best managed by:

a. Performing chest compressions
b. Administering epinephrine and sodium bicarbonate
c. Performing vigorous stimulation
d. Providing continuous positive pressure ventilation

A

a. Performing chest compressions

Rationale: When a newborn’s heart rate drops below 60 beats per minute despite adequate positive pressure ventilation, the appropriate response is to perform chest compressions to improve circulation and cardiac output. Administering epinephrine and sodium bicarbonate (option b) can be considered if compressions and ventilation are insufficient. Performing vigorous stimulation (option c) and providing continuous positive pressure ventilation (option d) are not as effective in this situation as chest compressions.

79
Q

A baby was born small for gestational age (SGA), with opacities of both eyes and a grade 3/6 machinery-like murmur. The most likely diagnosis is:
a. Congenital toxoplasmosis
b. Congenital herpes infection
c. Congenital rubella syndrome
d. Congenital syphilis

A

c. Congenital rubella syndrome

Rationale: Congenital rubella syndrome is characterized by intrauterine growth restriction (SGA), cataracts or other eye abnormalities (opacities), and congenital heart defects, such as a patent ductus arteriosus (which produces a machinery-like murmur). Congenital toxoplasmosis (option a) usually presents with chorioretinitis and intracranial calcifications, not a machinery-like murmur. Congenital herpes infection (option b) and congenital syphilis (option d) do not typically present with these specific features.

80
Q

Which of the following statements regarding late-onset neonatal sepsis is NOT correct?
a. It is common after one week of life until 60 days
b. It is either community or hospital-acquired infection
c. Meningitis and NEC are common complications
d. It is usually caused by an E. coli infection

A

d. It is usually caused by an E. coli infection

Rationale: Late-onset neonatal sepsis, occurring after the first week of life until 60 days, is commonly caused by coagulase-negative staphylococci, Staphylococcus aureus, and Gram-negative bacteria like Klebsiella and Pseudomonas. E. coli is more commonly associated with early-onset sepsis. The other statements are correct: late-onset sepsis can be community or hospital-acquired (option b), and complications like meningitis and necrotizing enterocolitis (NEC) are common (option c).

81
Q

The most common etiologic agent for early-onset neonatal sepsis is:
a. E. coli
b. Listeria monocytogenes
c. Pseudomonas aeruginosa
d. Staphylococcus aureus

A

a. E. coli

Rationale: Early-onset neonatal sepsis, occurring within the first week of life, is most commonly caused by Group B Streptococcus (GBS) and E. coli. While GBS is often considered the leading cause, E. coli is also a significant pathogen. Listeria monocytogenes (option b), Pseudomonas aeruginosa (option c), and Staphylococcus aureus (option d) are less common causes.

82
Q

The neonate may acquire infection through the placenta. This organism is rarely transmitted through the placenta:
a. Toxoplasmosis
b. Herpes simplex
c. Cytomegalovirus
d. Congenital syphilis

A

b. Herpes simplex

Rationale: Herpes simplex virus (HSV) is typically transmitted during delivery through direct contact with infected genital secretions. In contrast, Toxoplasmosis (option a), Cytomegalovirus (option c), and Congenital syphilis (option d) are known to be transmitted transplacentally.

83
Q

Baby Girl A was delivered full term, 40 weeks, with a birth weight of 1.5 kg. She has chorioretinitis, hepatomegaly, thrombocytopenia, cerebral calcification, and hydrocephalus. What is the most likely diagnosis?
a. Congenital toxoplasmosis
b. Congenital hepatitis B
c. Congenital herpes simplex
d. Congenital rubella

A

a. Congenital toxoplasmosis

Rationale: Congenital toxoplasmosis is characterized by chorioretinitis, hepatomegaly, thrombocytopenia, cerebral calcifications, and hydrocephalus. These findings are specific to toxoplasmosis, while the other conditions (congenital hepatitis B, congenital herpes simplex, and congenital rubella) do not present with this specific combination of symptoms.

84
Q

A newborn is examined with a gastrointestinal problem. An upper GI series is performed showing abnormal positioning of the ligament of Treitz and the cecum. Which of the following is most likely?
a. GI malrotation
b. Gastroesophageal reflux
c. Pyloric stenosis
d. Hirschsprung disease

A

a. GI malrotation

Rationale: Gastrointestinal malrotation is identified by the abnormal positioning of the ligament of Treitz and the cecum on an upper GI series. Gastroesophageal reflux (option b), pyloric stenosis (option c), and Hirschsprung disease (option d) present with different clinical and imaging findings.

85
Q

What is the most common cause of neonatal unconjugated hyperbilirubinemia?
a. Hemolytic disorders
b. Bacterial sepsis
c. Extrahepatic obstruction
d. Genetic disorders (e.g., Rotor syndrome, Dubin-Johnson syndrome)

A

b. Bacterial sepsis

86
Q

A 1-day-old infant who was born by a difficult forceps delivery is alert and active but does not move her left arm, which she keeps internally rotated by her side with the forearm extended and pronated; Moro reflex was likewise absent on the left arm. The rest of her physical examination is normal. This clinical picture most likely indicates:
a. Fracture of the left clavicle
b. Fracture of the left humerus
c. Erb’s palsy on the left arm
d. Phrenic nerve paralysis

A

c. Erb’s palsy on the left arm

Rationale: Erb’s palsy, a type of brachial plexus injury, is characterized by weakness or paralysis of the arm, with the arm held in an internally rotated position and the forearm extended and pronated. The absence of the Moro reflex on the affected side is also indicative of this condition. Fractures of the left clavicle (option a) or humerus (option b) would present differently, and phrenic nerve paralysis (option d) would affect diaphragmatic function.

87
Q

A 7-day-old infant is seen in the ER for fever and poor feeding. The baby was delivered vaginally 2 hours after the mother arrived at the hospital, at 36 weeks of gestation with a birth weight of 2.9 kg. Maternal lab tests were negative. The most likely organism causing these infant symptoms is:
a. Staphylococcus aureus
b. Listeria monocytogenes
c. Group B Streptococcus (GBS)
d. Streptococcus pneumoniae

A

c. Group B Streptococcus (GBS)

Rationale: Group B Streptococcus (GBS) is the most likely cause of early-onset neonatal sepsis in a baby presenting with fever and poor feeding within the first week of life, particularly in a preterm infant. Staphylococcus aureus (option a), Listeria monocytogenes (option b), and Streptococcus pneumoniae (option d) are less common causes.

88
Q

The best way to prevent the complications associated with prematurity is:
a. Administration of antenatal steroids
b. Adequate nutrition
c. Maternal multivitamin supplement
d. Aggressive neonatal resuscitation

A

a. Administration of antenatal steroids

Rationale: Administration of antenatal steroids is the best way to prevent complications associated with prematurity, such as respiratory distress syndrome, intraventricular hemorrhage, and necrotizing enterocolitis. Adequate nutrition (option b), maternal multivitamin supplements (option c), and aggressive neonatal resuscitation (option d) are important but do not specifically prevent complications of prematurity as effectively as antenatal steroids.

89
Q

Neonatal jaundice is pathologic in all of the following conditions EXCEPT:
a. Clinical jaundice in the first 24 hours of life
b. If total serum bilirubin increases by 5 mg/dL/day
c. If direct serum bilirubin or B2 exceeds 1.5-2 mg/dL (>10% of total serum bilirubin)
d. If clinical jaundice appears at 1 week or less in full-term infants or 2 weeks or less in preterm infants

A

d. If clinical jaundice appears at 1 week or less in full-term infants or 2 weeks or less in preterm infants

Rationale: Neonatal jaundice is considered pathologic if it presents within the first 24 hours of life (option a), if total serum bilirubin increases by more than 5 mg/dL per day (option b), or if direct serum bilirubin exceeds 1.5-2 mg/dL (option c). However, clinical jaundice appearing at 1 week or less in full-term infants or 2 weeks or less in preterm infants (option d) is not typically considered pathologic and can be part of the normal physiologic jaundice process.

90
Q

A 30-hour-old full-term infant has facial and chest jaundice. He is breastfeeding well and has an otherwise normal examination. His bilirubin level is 15.5 mg/dL. Which of the following is the most appropriate course of action?
a. Start phototherapy
b. Start an exchange transfusion
c. Recommend stopping breastfeeding for 48 hours and supplementing with milk formula
d. No action is needed

A

a. Start phototherapy

Rationale: For a 30-hour-old full-term infant with a bilirubin level of 15.5 mg/dL, phototherapy is indicated to reduce bilirubin levels and prevent the risk of kernicterus. Exchange transfusion (option b) is reserved for more severe cases with higher bilirubin levels or if phototherapy is ineffective. Recommending stopping breastfeeding (option c) is not necessary, and no action (option d) would not be appropriate given the elevated bilirubin level.

91
Q

Anemia of prematurity can be best described as:
a. It occurs in premature infants in the 5th month of life
b. It is caused by shortened RBC survival
c. Packed RBC transfusion is not indicated if the baby is asymptomatic
d. Adequate erythropoietin levels

A

b. It is caused by shortened RBC survival

Rationale: Anemia of prematurity is primarily due to shortened red blood cell (RBC) survival and a decreased production of erythropoietin. It typically occurs in premature infants in the early weeks of life, not the 5th month (option a). Packed RBC transfusion may be indicated if the baby is symptomatic (option c), and erythropoietin levels are usually inadequate in these infants (option d).

92
Q

Hyperbilirubinemia associated with Crigler-Najjar syndrome type 1 is caused by which of the following?
a. Severe deficiency of uridine diphosphate glucuronyltransferase
b. Increased production of bilirubin
c. Increased conjugation of bilirubin
d. Deficient hepatic uptake of bilirubin

A

a. Severe deficiency of uridine diphosphate glucuronyltransferase

Rationale: Crigler-Najjar syndrome type 1 is a rare genetic disorder characterized by a severe deficiency of the enzyme uridine diphosphate glucuronyltransferase (UGT), leading to an inability to conjugate bilirubin. This results in severe unconjugated hyperbilirubinemia. Increased production of bilirubin (option b), increased conjugation of bilirubin (option c), and deficient hepatic uptake of bilirubin (option d) are not the causes of hyperbilirubinemia in this condition.

93
Q

Which of the following decreases the risk of neurologic damage in a jaundiced newborn?
a. Maternal ingestion of phenobarbital during pregnancy
b. Hypoalbuminemia
c. Acidosis
d. Sepsis

A

a. Maternal ingestion of phenobarbital during pregnancy

Rationale: Maternal ingestion of phenobarbital during pregnancy can induce hepatic enzymes in the fetus, leading to increased bilirubin conjugation and thereby decreasing the risk of neurologic damage due to hyperbilirubinemia. Hypoalbuminemia (option b), acidosis (option c), and sepsis (option d) all increase the risk of bilirubin crossing the blood-brain barrier and causing neurologic damage.

94
Q

Baby G was brought to the ER on the 2nd week of life because of yellowish discoloration of the skin. He was lethargic with opisthotonus and spasticity. Total bilirubin was 30 mg/dL. The most likely diagnosis is:
a. Sepsis neonatorum
b. Perinatal asphyxia
c. Bilirubin encephalopathy
d. Persistent pulmonary hypertension

A

c. Bilirubin encephalopathy

Rationale: Bilirubin encephalopathy (kernicterus) is a condition resulting from very high levels of unconjugated bilirubin, which can cross the blood-brain barrier and cause neurological damage. Symptoms include lethargy, opisthotonus, and spasticity. Sepsis neonatorum (option a), perinatal asphyxia (option b), and persistent pulmonary hypertension (option d) do not specifically present with the described symptoms and bilirubin levels.

95
Q

ABO incompatibility can be differentiated from Rh incompatibility in that the former is:
a. Milder, hydrops is not common
b. Mother is Rh positive, baby is Rh negative
c. Weakly positive on Coombs’ test
d. Occurs during the 2nd pregnancy

A

a. Milder, hydrops is not common

Rationale: ABO incompatibility typically presents as a milder form of hemolytic disease compared to Rh incompatibility, and hydrops fetalis is not common. Rh incompatibility (option b) involves an Rh-negative mother and an Rh-positive baby. ABO incompatibility can result in a weakly positive Coombs’ test (option c), but this is not a key differentiator. It can occur during the first pregnancy, not just the second (option d).

96
Q

In treating hyperbilirubinemia, phototherapy can sometimes cause dark, grayish discoloration of the skin described as:
a. Gangrene
b. Bronze baby
c. Jaundice
d. Kernicterus

A

b. Bronze baby

Rationale: Phototherapy can occasionally cause a condition known as “bronze baby syndrome,” characterized by a dark, grayish discoloration of the skin. Gangrene (option a), jaundice (option c), and kernicterus (option d) are not associated with this specific skin discoloration due to phototherapy.

97
Q

Baby Boy M was born full term. Labor was uneventful. He was given breast milk. He was apparently well when he was noted to have yellowish discoloration of the skin on the 30th hour of life. No associated symptoms were noted. Total serum bilirubin level was 8 mg/dL. What is your impression?
a. Pathologic jaundice
b. Physiologic jaundice
c. Neonatal sepsis
d. G6PD deficiency

A

b. Physiologic jaundice

Rationale: Physiologic jaundice is common in newborns and typically appears after the first 24 hours of life, peaking around the third to fourth day. A bilirubin level of 8 mg/dL at 30 hours is within the expected range for physiologic jaundice. Pathologic jaundice (option a) would present within the first 24 hours or with much higher levels of bilirubin. Neonatal sepsis (option c) and G6PD deficiency (option d) would typically have other associated symptoms or risk factors.

98
Q

A 1-month-old baby was brought to a pediatrician because of hypotonia and constipation. Birth history was uncomplicated. The mother also noted persistence of an umbilical hernia and protrusion of the tongue. Which of the following metabolic disorders would you consider?
a. G6PD deficiency
b. Congenital hypothyroidism
c. Hypoglycemia
d. Phenylketonuria

A

b. Congenital hypothyroidism

Rationale: Congenital hypothyroidism presents with symptoms such as hypotonia, constipation, umbilical hernia, and macroglossia (protrusion of the tongue). G6PD deficiency (option a) typically presents with jaundice or hemolytic anemia. Hypoglycemia (option c) would present with symptoms related to low blood sugar levels. Phenylketonuria (option d) presents with developmental delays and a musty odor but not the constellation of symptoms described.

99
Q

Polycythemia is a neonatal complication seen in:
a. Placenta previa
b. Abruptio placenta
c. Delayed clamping of the umbilical cord
d. Donor twin

A

c. Delayed clamping of the umbilical cord

Rationale: Delayed clamping of the umbilical cord can lead to an increased red blood cell mass, resulting in polycythemia. Placenta previa (option a) and abruptio placenta (option b) are more likely to cause anemia due to bleeding. The donor twin in twin-to-twin transfusion syndrome (option d) typically has anemia rather than polycythemia.

100
Q

Baby A was born full term via Cesarean section. He developed rapid respiration in the 3rd hour of his life. Chest X-ray showed widened intercostal spaces and fluid in interlobar fissures. The most likely diagnosis is:
a. Neonatal pneumonia
b. Transient tachypnea of the newborn
c. Bronchopulmonary dysplasia
d. Respiratory distress syndrome

A

b. Transient tachypnea of the newborn

Rationale: Transient tachypnea of the newborn (TTN) is characterized by rapid breathing, usually within the first few hours of life, and is commonly seen in babies delivered via Cesarean section. Chest X-ray findings include fluid in the interlobar fissures and widened intercostal spaces. Neonatal pneumonia (option a), bronchopulmonary dysplasia (option c), and respiratory distress syndrome (option d) have different presentations and X-ray findings.

101
Q

A term baby was delivered in respiratory distress with a note of drooling saliva and difficulty inserting an orogastric tube during suctioning of oral secretions. Chest X-ray showed coiling of the feeding tube in the upper esophagus. The infant is suspected to have:
a. Pyloric stenosis
b. Intestinal obstruction
c. Congenital diaphragmatic hernia
d. Tracheoesophageal fistula

A

d. Tracheoesophageal fistula

Rationale: Tracheoesophageal fistula is suggested by respiratory distress, drooling, and the inability to pass an orogastric tube, with the tube coiling in the upper esophagus on X-ray. Pyloric stenosis (option a) presents later with projectile vomiting. Intestinal obstruction (option b) and congenital diaphragmatic hernia (option c) would have different clinical and radiographic findings.

102
Q

A premature infant, 28 weeks by gestational age, developed abdominal distension on the 10th day of life with feeding intolerance and bloody stools. Which of the following is the most likely diagnosis?
a. Hypertrophic pyloric stenosis
b. Intestinal obstruction
c. Tracheoesophageal fistula
d. Necrotizing enterocolitis

A

d. Necrotizing enterocolitis

Rationale: Necrotizing enterocolitis (NEC) is a serious gastrointestinal condition that typically affects premature infants, characterized by abdominal distension, feeding intolerance, and bloody stools. Hypertrophic pyloric stenosis (option a) presents later with projectile vomiting. Intestinal obstruction (option b) would present with vomiting and lack of stool passage, and tracheoesophageal fistula (option c) would present with respiratory issues and feeding difficulties soon after birth.

103
Q

A radiographic finding that is pathognomonic of necrotizing enterocolitis (NEC) is:
a. Double bubble sign
b. Pneumatosis intestinalis
c. Absence of presacral gas
d. Thickened bowel loops

A

b. Pneumatosis intestinalis

Rationale: Pneumatosis intestinalis, the presence of gas within the bowel wall, is pathognomonic for necrotizing enterocolitis (NEC). The double bubble sign (option a) is seen in duodenal atresia. Absence of presacral gas (option c) and thickened bowel loops (option d) are not specific to NEC.

104
Q

Which of the following is suggestive of congenital infection?
a. Increasing IgM titer
b. Decreasing IgM titer
c. Increasing IgG titer
d. Decreasing IgG titer

A

a. Increasing IgM titer

Rationale: An increasing IgM titer is suggestive of a congenital infection because IgM does not cross the placenta, so its presence in a newborn indicates that the infant has produced the antibody in response to an infection. Increasing IgG titer (option c) and decreasing IgG titer (option d) reflect maternal antibodies that cross the placenta and are not specific for congenital infection. Decreasing IgM titer (option b) is not indicative of active infection.

105
Q

Baby girl M., a term birth, 3 weeks old, was brought to you because of vomiting of previously ingested milk. The vomitus was described as non-bilious. On physical examination, she showed a moderate degree of dehydration, and a palpable olive-like mass was appreciated over the RUQ. The most likely diagnosis is:
a. Tracheoesophageal fistula
b. Congenital duodenal atresia
c. Hypertrophic pyloric stenosis
d. Congenital megacolon

A

c. Hypertrophic pyloric stenosis

Rationale: Hypertrophic pyloric stenosis typically presents in the first few weeks of life with non-bilious projectile vomiting, dehydration, and a palpable olive-like mass in the right upper quadrant. Tracheoesophageal fistula (option a) presents with feeding difficulties and respiratory symptoms. Congenital duodenal atresia (option b) typically presents with bilious vomiting. Congenital megacolon (option d), or Hirschsprung disease, presents with delayed passage of meconium and abdominal distension.

106
Q

Jaundice appearing between the second and third day after birth in full-term infants is due to:
a. Acute hemolytic jaundice
b. Physiologic jaundice
c. Biliary atresia
d. Neonatal hepatitis

A

b. Physiologic jaundice

Rationale: Physiologic jaundice is common and typically appears between the second and third day of life as the newborn’s liver matures and begins to process bilirubin. Acute hemolytic jaundice (option a) and neonatal hepatitis (option d) usually present earlier with more severe symptoms. Biliary atresia (option c) presents later with persistent jaundice.

107
Q

A newborn baby was noted to have yellowish hematocrit. Total bilirubin was 21 mg/dL. The appropriate initial treatment option would be:
a. Exposure to phototherapy
b. Exchange transfusion immediately
c. Administration of IV immunoglobulin
d. Continue monitoring without intervention

A

a. Exposure to phototherapy

Rationale: For a newborn with a total bilirubin level of 21 mg/dL, phototherapy is the appropriate initial treatment to reduce bilirubin levels and prevent kernicterus. Exchange transfusion (option b) is considered if bilirubin levels are critically high or do not respond to phototherapy. Administration of IV immunoglobulin (option c) is used in specific cases like isoimmune hemolytic disease. Continuing monitoring without intervention (option d) is not appropriate given the high bilirubin level.

108
Q

Baby JM showed deepening jaundice after phototherapy started, and the repeat bilirubin determination increased to 30 mg/dL. What will be the next treatment plan for this baby?
a. Continue exposure to phototherapy
b. Double volume exchange transfusion
c. Metalloporphyrin
d. Phenobarbital

A

b. Double volume exchange transfusion

Rationale: When bilirubin levels continue to rise despite phototherapy, reaching a critical level like 30 mg/dL, a double volume exchange transfusion is indicated to rapidly reduce bilirubin levels and prevent kernicterus. Continuing phototherapy (option a) alone is insufficient. Metalloporphyrin (option c) and phenobarbital (option d) are not immediate treatments for critically high bilirubin levels.

109
Q

Maternal drug addiction commonly results in:
a. Normal birth weight
b. Neonatal drug withdrawal
c. Large for gestational age (LGA)
d. No significant effect to the fetus

A

b. Neonatal drug withdrawal

Rationale: Maternal drug addiction commonly leads to neonatal drug withdrawal, also known as neonatal abstinence syndrome (NAS), which can present with symptoms such as irritability, feeding difficulties, and tremors. Normal birth weight (option a) and large for gestational age (option c) are less common. There is a significant impact on the fetus (option d).

110
Q

Neonatal thyrotoxicosis is common among mothers with:
a. Graves’ disease
b. Hypothyroidism
c. Hashimoto’s thyroiditis
d. Postpartum thyroiditis

A

a. Graves’ disease

Rationale: Neonatal thyrotoxicosis is most commonly seen in infants of mothers with Graves’ disease due to the transplacental transfer of thyroid-stimulating antibodies. Hypothyroidism (option b), Hashimoto’s thyroiditis (option c), and postpartum thyroiditis (option d) are not typically associated with neonatal thyrotoxicosis.

111
Q

The likelihood of having a baby with hyaline membrane disease (RDS) is reduced in pregnancies associated with:
a. C-section delivery
b. Diabetes mellitus
c. Chronic hypertension
d. Premature labor

A

c. Chronic hypertension

112
Q

Which of the following statements about congenital duodenal obstruction is true?
a. Abdominal X-ray will show “double-bubble” sign
b. Abdominal distension is more prominent
c. Non-bilious vomiting
d. Sawtooth contractions of the aganglionic segment

A

a. Abdominal X-ray will show “double-bubble” sign

Rationale: The “double-bubble” sign on an abdominal X-ray is characteristic of congenital duodenal obstruction. Abdominal distension (option b) is more prominent in distal obstructions. Non-bilious vomiting (option c) is not typical for duodenal atresia, which usually causes bilious vomiting. Sawtooth contractions (option d) are not related to duodenal obstruction but are associated with Hirschsprung disease.

113
Q

A 3-day-old baby was brought to the emergency room because of stooling since birth. This was associated with bilious vomiting and abdominal distension. All of the following laboratory exams should be done EXCEPT:
a. Plain abdomen X-ray
b. Barium enema
c. Gastroscopy
d. Rectal biopsy

A

c. Gastroscopy

Rationale: For a 3-day-old baby with bilious vomiting and abdominal distension, appropriate diagnostic tests include a plain abdominal X-ray (option a), barium enema (option b), and rectal biopsy (option d) to evaluate for potential causes like intestinal obstruction or Hirschsprung disease. Gastroscopy (option c) is not typically indicated in the initial evaluation of these symptoms.

114
Q

A 4-day-old newborn baby was noted to have not passed meconium since birth. There was abdominal distension and referred to a pediatric surgeon. Hirschsprung disease was considered. What is the pathology in this condition?
a. Absence of ganglion cells in the bowel
b. Extensive bowel necrosis
c. Hypertrophy of the muscles of the pylorus
d. Malrotation of the intestines

A

a. Absence of ganglion cells in the bowel

Rationale: Hirschsprung disease is characterized by the absence of ganglion cells in the bowel, leading to a lack of peristalsis and subsequent bowel obstruction. Extensive bowel necrosis (option b) is not specific to Hirschsprung disease. Hypertrophy of the muscles of the pylorus (option c) is seen in hypertrophic pyloric stenosis. Malrotation of the intestines (option d) is a different condition involving abnormal positioning of the intestines.

115
Q

A newborn infant has positive Coomb’s test, spherocytes, increased bilirubin, and increased reticulocytes. Which of the following is most likely?
a. Anemia of prematurity
b. G6PD deficiency
c. Autoimmune hemolytic anemia
d. Intraventricular hemorrhage

A

c. Autoimmune hemolytic anemia

Rationale: A positive Coombs test, spherocytes, increased bilirubin, and increased reticulocytes are indicative of autoimmune hemolytic anemia. Anemia of prematurity (option a) typically involves low reticulocyte counts and is not associated with a positive Coombs test. G6PD deficiency (option b) can cause hemolysis but is not associated with a positive Coombs test or spherocytes. Intraventricular hemorrhage (option d) is not related to these hematological findings.

116
Q

You are called to the NICU to evaluate a 3-hour-old term newborn born to a 20-year-old G1P1 mother. The pregnancy was uncomplicated, but meconium staining was noted at delivery. BW 3.4 kg, afebrile, HR 165 and RR 70 with clear breath sounds. O2 saturation at room air is 68%. O2 challenge test was done by giving 100% O2. After 5 minutes, the O2 saturation is 78%. The best explanation for the findings of the O2 challenge test is:

a. Pneumonia
b. Retained fetal lung fluid
c. Meconium aspiration
d. Cyanotic congenital heart disease

A

d. Cyanotic congenital heart disease

Rationale: In the context of meconium staining and significant hypoxemia that does not improve substantially with 100% oxygen, cyanotic congenital heart disease is the most likely diagnosis. Pneumonia (option a), retained fetal lung fluid (option b), and meconium aspiration (option c) could cause respiratory distress but are more likely to show improvement in oxygen saturation with 100% oxygen.

117
Q

A full-term infant was born by normal spontaneous delivery. Nursery stay was unremarkable and the baby was breastfed. On the 2nd week of life, the baby was noted to be jaundiced. This baby is most likely having:
a. Kernicterus
b. Pathologic jaundice
c. Physiologic jaundice
d. Breastmilk jaundice

A

d. Breastmilk jaundice

Rationale: Breastmilk jaundice typically presents after the first week of life in an otherwise healthy, breastfed infant. Kernicterus (option a) is associated with severe hyperbilirubinemia and neurological symptoms. Pathologic jaundice (option b) would usually present earlier. Physiologic jaundice (option c) typically resolves by the second week.

118
Q

The pathophysiology of hyaline membrane disease appears to involve:
a. Increased metabolism of pulmonary surfactant
b. Decreased metabolism of pulmonary surfactant
c. Increased production of pulmonary surfactant
d. Decreased production of pulmonary surfactant

A

d. Decreased production of pulmonary surfactant

Rationale: Hyaline membrane disease (respiratory distress syndrome) is caused by decreased production of pulmonary surfactant, leading to alveolar collapse and respiratory distress. Increased metabolism of pulmonary surfactant (option a) and increased production of pulmonary surfactant (option c) are incorrect. Decreased metabolism of pulmonary surfactant (option b) is not a contributing factor.

119
Q

Peripheral pulmonic stenosis, atrial septal defect, ventricular septal defect, chorioretinitis, hepatosplenomegaly, jaundice, and “blueberry muffin spots” are the clinical manifestations typically associated with which congenital infection?
a. Rubella
b. Cytomegalovirus (CMV)
c. Toxoplasmosis
d. Syphilis

A

a. Rubella

Rationale: Congenital rubella syndrome includes clinical manifestations such as peripheral pulmonic stenosis, atrial septal defect, ventricular septal defect, chorioretinitis, hepatosplenomegaly, jaundice, and “blueberry muffin spots.” Cytomegalovirus (option b), toxoplasmosis (option c), and syphilis (option d) present with different sets of symptoms.

120
Q

Which of the following statements regarding clavicular fracture in newborns is NOT correct?
a. Occurs frequently in infants of diabetic mothers
b. Requires treatment with a sling
c. Is associated with Erb’s palsy
d. Causes asymmetric Moro’s reflex

A

b. Requires treatment with a sling

Rationale: Clavicular fractures in newborns typically heal on their own without the need for a sling. It is commonly associated with Erb’s palsy (option c) and can cause an asymmetric Moro reflex (option d). It occurs frequently in infants of diabetic mothers (option a).

121
Q

Direct-reacting (conjugated) hyperbilirubinemia on the 10th day of life suggests one of the following:
a. G6PD deficiency
b. Neonatal hepatitis
c. Gilbert syndrome
d. Crigler-Najjar syndrome

A

b. Neonatal hepatitis

Rationale: Direct-reacting hyperbilirubinemia on the 10th day of life suggests neonatal hepatitis. G6PD deficiency (option a), Gilbert syndrome (option c), and Crigler-Najjar syndrome (option d) are associated with unconjugated hyperbilirubinemia.

122
Q

All of the following are expected problems of an infant of a recently diagnosed diabetic mother EXCEPT:
a. Hypoglycemia
b. Transient tachypnea of the newborn
c. Intrauterine growth restriction
d. Perinatal asphyxia

A

c. Intrauterine growth restriction

Rationale: Infants of diabetic mothers are at risk for hypoglycemia (option a), transient tachypnea of the newborn (option b), and perinatal asphyxia (option d). However, they are more likely to be large for gestational age rather than experiencing intrauterine growth restriction.

123
Q

Which commonly abused drug, produced in illegal laboratories and popular among adolescents and young adults because of its potency and ease of absorption?
a. Methamphetamines
b. Cocaine
c. Ecstasy (MDMA)
d. LSD

A

a. Methamphetamines

Rationale: Methamphetamines are commonly produced in illegal laboratories and are popular among adolescents and young adults due to their potency and ease of absorption. Cocaine (option b) and Ecstasy (MDMA) (option c) are also abused but are not primarily produced in illegal laboratories. LSD (option d) is a hallucinogen and not produced for the same reasons as methamphetamines.

124
Q

A term male infant is delivered vaginally to a 22-year-old mother. Immediately after birth, the baby has a scaphoid abdomen, cyanosis, and respiratory distress. Heart sounds are heard on the right side of the chest and diminished breath sounds on the left side. What is your next course of action?
a. Initiate bag-mask ventilation
b. Administer Naloxone
c. Administer Sodium bicarbonate
d. Intubate with an endotracheal tube

A

d. Intubate with an endotracheal tube

Rationale: The presentation suggests a congenital diaphragmatic hernia, which requires immediate intubation to secure the airway and avoid bag-mask ventilation (option a), which could exacerbate the condition. Administering Naloxone (option b) and Sodium bicarbonate (option c) are not appropriate for this condition.

125
Q

Which of the following is NOT a symptom of hypoglycemia?
a. Jitteriness
b. Vomiting
c. Cyanosis
d. Hypothermia

A

b. Vomiting

Rationale: Vomiting is not a typical symptom of hypoglycemia. Symptoms of hypoglycemia include jitteriness (option a), cyanosis (option c), and hypothermia (option d).

126
Q

A term baby boy born to a mother with IDDM has a bedside HGT of 32 mg/dL at 1 hour of life. He is awake and has normal vital signs. Which of the following is the appropriate next step in management?
a. Administer a dose of Dextrose
b. Total serum bilirubin and calcium
c. Instruct the mother to breastfeed early and recheck the glucose after 30 minutes
d. Recheck the glucose in 1 hour

A

c. Instruct the mother to breastfeed early and recheck the glucose after 30 minutes

Rationale: Early breastfeeding and rechecking glucose levels after 30 minutes is appropriate to address mild hypoglycemia in a stable, asymptomatic infant. Administering a dose of Dextrose (option a) is reserved for more severe or symptomatic hypoglycemia. Total serum bilirubin and calcium (option b) are not immediately relevant. Rechecking the glucose in 1 hour (option d) is too long to wait.

127
Q

A term baby weighing 4.3 kg is born without complication to a mother with gestational diabetes. At 12 hours of life, he appears mildly jaundiced. Vital signs are stable, he is feeding well, and his blood type is the same as the mother’s. Which of the following laboratory tests are most likely to help you evaluate this infant’s jaundice?
a. Total bilirubin and glucose
b. Total serum bilirubin and hematocrit
c. Total serum bilirubin, liver transaminases, and hepatitis panel
d. Total serum bilirubin and calcium

A

b. Total serum bilirubin and hematocrit

Rationale: Total serum bilirubin and hematocrit are appropriate to evaluate the extent of jaundice and to assess for polycythemia, which is common in infants of diabetic mothers. Total bilirubin and glucose (option a), total serum bilirubin, liver transaminases, and hepatitis panel (option c), and total serum bilirubin and calcium (option d) are not the most relevant initial tests.

128
Q

A 12-hour-old infant who has been feeding poorly becomes tachypneic with grunting. Which of the following lab tests has the lowest diagnostic yield?
a. Blood culture
b. Urine culture
c. CBC
d. Chest x-ray

A

b. Urine culture

Rationale: A urine culture has the lowest diagnostic yield in a 12-hour-old infant with respiratory distress. Blood culture (option a), CBC (option c), and chest X-ray (option d) are more relevant for diagnosing conditions like sepsis or respiratory distress syndrome.

129
Q

An infant is born at 36 weeks’ gestation to a 30-year-old G3P3 mother by spontaneous vaginal delivery. Rupture of membranes occurred 15 hours prior to delivery. BW is 400 grams, and Apgar scores are 6 and 9 at 1 and 5 minutes respectively. Which of the following factors places this baby at greatest risk for neonatal sepsis?
a. Gestational age
b. Apgar score
c. Maternal age
d. Length of time membranes were ruptured

A

a. Gestational age

Rationale: Being born at 36 weeks’ gestation places the infant in the late preterm category, which is a significant risk factor for neonatal sepsis. Late preterm infants have an immature immune system and are more susceptible to infections compared to full-term infants. While prolonged rupture of membranes (more than 18 hours) is also a risk factor, in this case, the gestational age (prematurity) is the primary concern. Apgar scores, although indicative of the infant’s initial condition, are less specific for sepsis risk. Maternal age is generally not a significant risk factor for neonatal sepsis.

130
Q

A 13-year-old boy is brought to an adolescent clinic by his mother because she is concerned that her son has not developed any signs of puberty. She recalls that her older daughter began pubertal development when she was younger than her son now. Which of the following statements about normal sequence of pubertal development in males and females is correct?
a. Puberty begins in males approximately 12-18 months later than in females
b. Menarche occurs at the same time as thelarche
c. Pubic hair growth is the first sign of puberty in males
d. Pubic hair growth is the first sign of puberty in females

A

a. Puberty begins in males approximately 12-18 months later than in females

Rationale: Puberty typically begins in males about 12-18 months later than in females. Menarche (onset of menstruation) (option b) does not occur at the same time as thelarche (breast development). Pubic hair growth is not the first sign of puberty in males (option c) or females (option d).

131
Q

Adolescents are emotional, impulsive, and thrill-seeking because there is a later development of the pre-frontal cortex which is in charge of:
a. Emotions and motivations
b. Physical coordination and reward
c. Judgment and present orientation
d. Impulse control and planning

A

d. Impulse control and planning

Rationale: The pre-frontal cortex is responsible for impulse control and planning, which develops later in adolescence. Emotions and motivations (option a) are associated with the limbic system. Physical coordination and reward (option b) involve other brain areas. Judgment and present orientation (option c) are also functions of the pre-frontal cortex but are less specific than impulse control and planning.

132
Q

The following are risk factors for the risk-taking behaviors of adolescents EXCEPT:
a. Violence at home
b. Low socio-economic status
c. Single parent household
d. Religious involvement

A

d. Religious involvement

Rationale: Religious involvement is generally considered a protective factor against risk-taking behaviors in adolescents. Violence at home (option a), low socio-economic status (option b), and living in a single-parent household (option c) are risk factors for such behaviors.

133
Q

Adolescent thinking can be characterized by the following EXCEPT:
a. Prone to exaggeration and over-dramatization
b. Have the sense of invincibility
c. Self-centered
d. Altruistic

A

d. Altruistic

Rationale: Adolescents are often prone to exaggeration and over-dramatization (option a), have a sense of invincibility (option b), and can be self-centered (option c). While some adolescents can certainly be altruistic, it is not a characteristic that typically defines adolescent thinking as much as the other listed traits.

134
Q

The following are included in the Top 10 causes of morbidity and mortality of adolescents based on Department of Health Data EXCEPT:
a. Injury
b. Leukemia
c. Alcohol overdose
d. Homicide

A

c. Alcohol overdose

Rationale: Common causes of morbidity and mortality in adolescents include injury (option a), leukemia (option b), and homicide (option d). While alcohol use is a significant health issue, alcohol overdose (option c) is not typically listed among the top 10 causes of morbidity and mortality in adolescents.

135
Q

Some of the common risk behaviors in adolescents include the use of alcohol, tobacco, and drugs. In the country, there are:
a. Drug users > smokers > alcohol drinkers
b. Smokers > alcohol drinkers > drug users
c. Alcohol drinkers > smokers > drug users
d. Smokers > drug users > alcohol drinkers

A

c. Alcohol drinkers > smokers > drug users

Rationale: In many countries, including the Philippines, alcohol use is more common than smoking, and smoking is more common than drug use among adolescents.

136
Q

Which among the following statements regarding violence is true:
a. Cyberbullying is very common, but is never disclosed or reported among the youth.
b. Media decreases the risk of violence among the youth.
c. There are more youth who admit to hurting someone than being hurt by someone in the past 12 months.
d. Family and the environment are the only factors to consider in screening for violence.

A

c. There are more youth who admit to hurting someone than being hurt by someone in the past 12 months.

Rationale: Among the youth, there is often a higher self-reported incidence of hurting others than being hurt themselves. Cyberbullying (option a) is common and often underreported, but not “never” reported. Media exposure (option b) can increase the risk of violence, not decrease it. Family and environment (option d) are important, but not the only factors to consider in screening for violence.

137
Q

A 16-year-old female adolescent presents to the clinic with complaints of diffuse abdominal pain, fever, and nausea. On further questioning, she admits she is sexually active and most of the time uses condoms for protection. Pelvic examinations show moderate lower abdominal tenderness and tenderness on palpation of her cervix and right ovary. No adnexal masses palpated. What is the next step in management of this patient?
a. Order wet mount saline microscopy to demonstrate motile protozoa
b. Prescribe oral doxycycline for 2 weeks
c. Outpatient treatment with a single dose of ofloxacin
d. Admit patient and start IV cefoxitin and oral doxycycline

A

d. Admit patient and start IV cefoxitin and oral doxycycline

Rationale: The patient’s symptoms and physical findings suggest pelvic inflammatory disease (PID). Given the severity of her symptoms, including fever and significant abdominal pain, the appropriate next step is to admit her for inpatient treatment with IV antibiotics (IV cefoxitin) and oral doxycycline. Outpatient treatment (options b and c) might be appropriate for less severe cases. Wet mount saline microscopy (option a) is not the immediate next step and is more specific for diagnosing Trichomonas infection.

138
Q

A 16-year-old female adolescent presents to the clinic with complaints of diffuse abdominal pain, fever, and nausea. On further questioning, she admits she is sexually active during the past year and most of the time uses condoms for protection. Pelvic examinations show moderate lower abdominal tenderness and tenderness on palpation of her cervix and right ovary. No adnexal masses palpated. What is the next step in management of this patient?
a. Order wet mount saline microscopy to demonstrate motile protozoa
b. Prescribe oral doxycycline for 2 weeks
c. Outpatient treatment with a single dose of ofloxacin
d. Admit patient and start IV cefoxitin and oral doxycycline

A

d. Admit patient and start IV cefoxitin and oral doxycycline

Rationale: The patient’s symptoms suggest pelvic inflammatory disease (PID) with moderate severity (fever, significant abdominal pain, cervical and adnexal tenderness). Hospital admission for intravenous antibiotics (IV cefoxitin and oral doxycycline) is appropriate to ensure adequate treatment and monitoring. Outpatient treatment (options b and c) might be appropriate for less severe cases. Wet mount saline microscopy (option a) is used for diagnosing Trichomonas infection, not for immediate management of PID.

139
Q

Which of the following factors are impacts of early teen pregnancy on the fetus and newborn?
a. Low birth weight, Malnutrition, Pre-eclampsia
b. Fetal mortality, Low birth weight, Malnutrition
c. Malnutrition, Fetal mortality, Neonatal pneumonia
d. Perinatal asphyxia, LGA

A

b. Fetal mortality, Low birth weight, Malnutrition

Rationale: Early teen pregnancy can negatively impact the fetus and newborn, leading to increased risks of fetal mortality, low birth weight, and malnutrition. Pre-eclampsia (option a) affects the mother more directly. Neonatal pneumonia (option c) and LGA (option d) are not primary impacts of teen pregnancy.

140
Q

In dealing with adolescents who present with risk-taking behavior, we must:
a. Ask the adolescents together with their parents regarding their risk-taking behavior
b. Do a complete sexual history even if they only came in with a chief complaint of cough
c. Disclose to the parents that their teen tried drinking a glass of beer for the first time in the last party s/he was allowed to attend
d. Skip the complete physical examination and just do a laboratory test if s/he admits to doing drugs

A

b. Do a complete sexual history even if they only came in with a chief complaint of cough

Rationale: It is important to take a complete sexual history as part of a thorough assessment for adolescents, even if they present with a seemingly unrelated complaint like a cough. Asking together with parents (option a) may inhibit the adolescent’s openness. Disclosing minor risk behaviors to parents without the adolescent’s consent (option c) can breach trust. Skipping the physical examination (option d) is not appropriate; a thorough assessment is necessary.

141
Q

Among adolescent girls presenting with pelvic pain, which life-threatening condition should be ruled out?
a. Abruptio placentae
b. Mittelschmerz
c. Ectopic pregnancy
d. Dysmenorrhea

A

c. Ectopic pregnancy

142
Q

The most common pelvic mass among adolescent girls is:
a. Functional cysts
b. Tuboovarian complex
c. Chocolate cyst
d. Dermoid cyst

A

a. Functional cysts

Rationale: Functional cysts are the most common type of pelvic mass found in adolescent girls. Tuboovarian complexes (option b), chocolate cysts (endometriomas) (option c), and dermoid cysts (option d) are less common.

143
Q

An 18-year-old came with a chief complaint of irregular menstruation, last having it 3-4 months ago. She had her menarche at 11 years old. The mother said she is not sexually active. On physical examination, you can feel a pelvic mass, but the rest of the physical examination is normal. Her irregular menstruation is due to:
a. Anovulation
b. Pregnancy
c. Polycystic ovarian syndrome
d. Malignancy

A

c. Polycystic ovarian syndrome

Rationale: Polycystic ovarian syndrome (PCOS) is characterized by irregular menstruation and the presence of a pelvic mass (enlarged ovaries with multiple cysts). Given the patient’s history and the physical finding of a pelvic mass, PCOS is the most likely diagnosis. Anovulation (option a) is a common feature of PCOS. Pregnancy (option b) is unlikely given the mother’s statement and the absence of other symptoms. Malignancy (option d) is less likely given the normal physical examination apart from the pelvic mass.

144
Q

A 12-year-old came with a chief complaint of irregular menstruation, having it every 2-3 months. She said she had her menarche 10 months ago and does not have any other complaints. Your physical examination was normal. Her irregular menstruation is due to:
a. Anovulation
b. Pregnancy
c. Polycystic ovarian syndrome
d. Mass/Malignancy

A

a. Anovulation

Rationale: Irregular menstruation in a 12-year-old who had menarche 10 months ago is likely due to anovulation, which is common in the first few years after menarche as the hypothalamic-pituitary-ovarian axis matures. Pregnancy (option b) and PCOS (option c) are less likely given the patient’s age and the normal physical examination. Mass/malignancy (option d) is unlikely without other symptoms or abnormal physical findings.

145
Q

Primary amenorrhea is the lack of menses by _ years old in the absence of secondary sexual characteristics.
a. 13
b. 14
c. 15
d. 16

A

a. 13

Rationale: Primary amenorrhea is defined as the absence of menses by age 13 in the absence of secondary sexual characteristics.

146
Q

Primary amenorrhea is the lack of menses by _ years old with normal secondary sexual characteristics development.
a. 13
b. 14
c. 15
d. 16

A

c. 15

Rationale: Primary amenorrhea is defined as the absence of menses by age 15 in the presence of normal secondary sexual characteristics.

147
Q

Testicular mass in an adolescent that needs to be ruled out since it is the most common testicular neoplasia:
a. Hydrocele
b. Varicocele
c. Spermatocele
d. Seminoma

A

d. Seminoma

Rationale: Seminoma is the most common type of testicular neoplasia in adolescents and needs to be ruled out when a testicular mass is present. Hydrocele (option a), varicocele (option b), and spermatocele (option c) are less likely to be neoplastic.

148
Q

A testicular mass characterized as a “bag of worms” appreciated on palpation that would transilluminate when a light is held behind the scrotum in a dark room is:
a. Hydrocele (transilluminates)
b. Varicocele (bag of worms)
c. Spermatocele
d. Seminoma

A

b. Varicocele

Rationale: A varicocele is described as a “bag of worms” on palpation and does not typically transilluminate. A hydrocele (option a) transilluminates but does not feel like a “bag of worms.” Spermatocele (option c) and seminoma (option d) have different presentations and characteristics.

149
Q

A 15-year-old male presents with bilateral breast enlargement and SMR 3 pubertal development. The rest of the physical examination is normal. Which of the following is the most appropriate next step in management?
a. Request for chromosomal analysis
b. Check serum estrogen level
c. Order liver function test
d. No test to be done and provide reassurance

A

d. No test to be done and provide reassurance

Rationale: Bilateral breast enlargement during puberty is common and usually benign (pubertal gynecomastia). Reassurance is often the appropriate management. Chromosomal analysis (option a), checking serum estrogen level (option b), and ordering liver function tests (option c) are not necessary unless there are other concerning findings or persistent symptoms.

150
Q

Suicide is the 3rd leading cause of death among 15-19 years of age after unintentional injuries and homicide. The risk factors for suicide include all except:
a. Substance abuse
b. Peer/family conflict
c. Divorce or separation of parents
d. Significant loss such as death of grandmother

A

d. Significant loss such as death of grandmother

Rationale: While significant loss, such as the death of a grandmother, can be a contributing factor, it is less directly associated with adolescent suicide compared to substance abuse (option a), peer/family conflict (option b), and divorce or separation of parents (option c).

151
Q

Adolescents may be reluctant to report rape for the following reasons EXCEPT:
a. Self-blame
b. Fear
c. Embarrassment
d. None of the above

A

d. None of the above

Rationale: Adolescents may indeed be reluctant to report rape due to self-blame (option a), fear (option b), and embarrassment (option c). Therefore, there is no exception among these reasons.

152
Q

A group of classmates brings a 16-year-old adolescent male to the ER after he becomes agitated, hyperactive, and hypersexual at an all-night rave. Which of the following substances is the boy most likely to have taken?
a. Ecstasy (MDMA)
b. LSD
c. Phencyclidine (PCP)
d. Methamphetamine

A

a. Ecstasy (MDMA)

Rationale: Ecstasy (MDMA) is known to cause agitation, hyperactivity, and hypersexual behavior. LSD (option b), PCP (option c), and methamphetamine (option d) can cause agitation and hyperactivity but are less associated with hypersexual behavior.

153
Q

A 16-year-old adolescent female complains of a nearly constant headache for the past year. There was no nausea or vomiting. She admits to hating school and does not participate in extracurricular activities and has difficulty in concentrating. Findings on neurologic examination are normal. What is the best next step in management?
a. Request a CT scan
b. Prescribe Sumatriptan
c. Keep a headache diary
d. Refer for psychosocial evaluation and counseling

A

d. Refer for psychosocial evaluation and counseling

Rationale: Given the context of chronic headaches, dislike for school, lack of participation in activities, and difficulty concentrating, a psychosocial evaluation and counseling is the best next step. A CT scan (option a) is not indicated without focal neurological findings. Prescribing Sumatriptan (option b) may not address the underlying issue. Keeping a headache diary (option c) can be useful but is not the most immediate step.

154
Q

A 15-year-old boy was found smoking marijuana by a teacher patrolling the school ground. The teacher noted that he has been acting “dazed and confused” and that his hygiene has worsened over the past few months. Which of the following would not be used as an assessment of substance abuse disorder in this teenager?
a. Driving while using marijuana
b. Problems with law enforcement
c. Craving marijuana use while at school
d. No longer attending soccer practice to smoke with friends

A

b. Problems with law enforcement

Rationale: Problems with law enforcement (option b) can be a result of substance abuse but do not directly indicate the pattern or severity of substance use itself. The other options, driving while using marijuana (option a), craving marijuana use while at school (option c), and no longer attending soccer practice to smoke with friends (option d), directly involve the use of the substance and can be used as criteria to assess substance abuse disorder. Thus, only option b does not directly assess the use of the substance.

155
Q

There are five stages of drug abuse in adolescents. What is the second stage of drug abuse in adolescents, where the primary focus is on reviewing risks of drug use and emphasizing safety?
a. No Drug Use
b. Seeking Euphoria
c. Experimentation
d. Seeking Preoccupation
e. Drug Use/Dependence

A

c. Experimentation

Rationale: The second stage of drug abuse in adolescents is experimentation, where adolescents begin to try drugs and the focus should be on reviewing risks and emphasizing safety. The other options do not correctly represent the second stage.

156
Q

Which of the following commonly abused drugs is a CNS stimulant that increases dopamine levels by preventing reuptake and may elicit euphoria, increased motor activity, decreased fatiguability, and mental alertness?
a. Amphetamines
b. Cocaine
c. Ecstasy (MDMA)
d. LSD

A

b. Cocaine

Rationale: Cocaine is a CNS stimulant that increases dopamine levels by preventing reuptake, leading to effects such as euphoria, increased motor activity, decreased fatiguability, and mental alertness. Amphetamines (option a) have similar effects but work through a different mechanism. Ecstasy (MDMA) (option c) is both a stimulant and hallucinogen. LSD (option d) is a hallucinogen.